ĐỊNH LÝ CAUCHY - DAVENPORT VÀ ỨNG DỤNG
3. Một số ứng dụng
Ví dụ 2 (Poland 2003). Chứng minh rằng với mọi số nguyên tốp > 3, tồn tại các số nguyên x; y; k sao cho0 < 2k < pvàx2Cy2 DkpC3.
Lời giải. Kết quả của bài toán không thay đổi nếu thayx; y bởip x; p y. Do đó ta sẽ chỉ ra các sốx; ytrong miền06 x; y < p
2 thỏa mãn điều kiện bài toán. Khix; y nằm trong miền này thì hiển nhiên phần thươngksẽ thỏa2k < p, vì
x2Cy2 < p2 4 C p2
4 D p2 2 :
ĐặtS là tập chứa tất cảbình phương theo modulop. Chú ý x2 .p x/2 .mod p/;8x 2 n
1; 2; : : : ;hp 2
io. Do đójSj D pC1
2 . Theo định lý 2.1 thì jSCSj>minfp; 2jSj 1g Dmin
p; 2:pC1
2 1
Dp ) jSCSj Dp:
Điều này chứng tỏ mọi thặng dư modulo p đều có thể viết được thành tổng của hai số chính phương. Và bài toán là trường hợp đặc biệt với kết quả bằng 3.
Ví dụ 3. Choplà số nguyên tố. Cho trướcp 1số nguyên sao cho không có số nào chia hết chop. Chứng minh rằng ta có thể đổi dấu một vài số trong chúng để tổng của các số thu được chia hết chop.
Lời giải. Gọip 1số nguyên làa1; a2; : : : ; ap 1. Với mỗii D1; 2; : : : ; p 1, đặt Ai D fai; aig .mod p/:
Khi đójAij D2;8i D1; 2; : : : ; p 1vìai 6 ai .mod p/. Áp dụng định lý 2.6 ta có jA1CA2C CAp 1j>minfp;jA1j C CAp 1 .p 2/g Dminfp; pg D p:
Từ đây suy ra
jA1CA2C CAp 1j Dp
hay tậpA1CA2 C CAp 1 chứa một hệ thặng dư đầy đủ modulop. Đây chính là kết luận của bài toán.
Ví dụ 4. Giả sử p > 2 là số nguyên tố và cho p 1 số nguyên a1; a2; : : : ; ap 1 thỏa mãn a1 ap 1 6 ::: p vàa1Ca2C Cap 1 6 ::: p. Chứng minh rằng có thể chia tập này thành hai nhóm rời nhau để tổng các phần tử của hai nhóm đồng dư với nhau theo modulop.
Lời giải. ĐặtAi D f0; aig .mod p/. Thì doai 6::: p;8i D1; 2; : : : ; p 1nênjAij D2;8i D 1; 2; : : : ; p 1. Theo định lý 2.6 thì
jA1CA2C CAp 1j>minfp;jA1jC CjAp 1j .p 2/g Dminfp; 2.p 1/ .p 2/g Dp:
Do đóA1CA2C CAp 1 DZp. Khi đó tồn tạibi 2 Ai; i D1; 2; : : : ; p 1sao cho b1Cb2C Cbp 1 a1Ca2C Cap 1
2 .modp/:
Vì mỗi phần tửbi 2 Aithìbi hoặc bằng 0, hoặc bằngai. Tuy nhiên doa1Ca2C Cap 1 60 .mod p/nên tồn tại một số phần tử bi khác 0. Gọi các phần tử đó làaj1; : : : ; ajk, thì thay vào ta có
aj1 C Cajk a1Ca2C Cap 1
2 .modp/
hay
2 aj1 C Cajk
a1C Cap 1 .mod p/
hay
aj1C Cajk .a1C Cap 1/ .aj1C Cajk/ .mod p/
bài toán được chứng minh.
Ví dụ 5 (USAMO 2009). Cho nlà số nguyên dương. Xác định tập con Alớn nhất của tập f n; nC1; : : : ; n 1; ngsao cho trongAkhôngtồn tại ba phần tửa; b; c (không nhất thiết phân biệt) màaCbCc D0.
Phân tích và hướng dẫn giải. Nhận xét062 A(vì nếu02Athì0D0C0C0, mâu thuẫn với tính chất của tậpA). Đặt
ACDA\ f1; 2; : : : ; ng; A DA\ f n; nC1; : : : ; 1g:
Nhận xétACCA và Alà hai tập conrời nhaucủaf n; nC1; : : : ; n 1; ng. Thật vậy, nếu.ACCA /\. A/ ¤ ;, khi đó tồn tại sốcmà
c DiCj D k .i 2 f1; 2; : : : ; ng\A; j 2 f n; nC1; : : : ; 1g\A; k2A/)iCjCk D0;
mâu thuẫn với định nghĩa của tậpA.
VìACCA và Alà hai tập con rời nhau của tậpf n; nC1; : : : ; n 1; ng (tập này có2nC1phần tử) nên
jACCA j C j„ƒ‚…Aj
DjAj
62nC1)2nC1>jACCA j C jAj:
Ngoài ra theo định lý 1.2 thì
jACCA j>jACj C jA j 1:
Do đó
2nC1>j„ ƒ‚ …ACj C jA j
DjAj
1C jAj D2jAj 1) jAj6nC1:
Đến đây dự đoán tậpAlớn nhất cónC1phần tử. Việc tiếp theo cần chỉ ra một tậpAcónC1 phần tử. Để thỏa mãn bài toán, ta chỉ cần lấy tập Acác phần tử lớn nhất và nhỏ nhất của tập đề bài. Đó là tập
AD 8ˆ ˆ<
ˆˆ
: n; nC1; : : : ; hn 2 i
„ ƒ‚ …1
phần âm
;hn 2
iC1; : : : ; n
„ ƒ‚ …
phần dương
9>
>=
>>
;:
Lấy ba phần tửa; b; c tùy ý của tậpAnày. Nếu cả ba phần tử cùng thuộc phần âm, hoặc phần dương của tập Athì hiển nhiênaCbCc ¤ 0. Nếu hai số thuộc phần dương, giả sửa; b, và một sốcthuộc phần âm, khi đóc > nvà
aCb >hn 2
iC1Chn 2
iC1D2hn
2 C1i
> 2 n
2 Dn>c )aCbCc > 0:
Tương tự kiểm tra nếu hai số thuộc phần âm, một số thuộc phần dương thì tậpAnày thỏa mãn đề bài.Trở ngại duy nhất còn lại là tậpAnày cónC1phần tử chỉ khinlẻ. Còn nếunchẵn, thì tậpAnày chỉ cónphần tử, không đạt được tớinC1phần tử. Tuy nhiên ta sẽ chỉ ra nếu nchẵn thì tậpAkhông thể cónC1phần tử được như dưới đây. Thật vậy, giả sửjAj DnC1 vẫn đúng vớinchẵn. DoA f n; : : : ; 1gvàAC f1; 2; : : : ; ngnên
ACCA f nC1; nC2; : : : ; n 2; n 1g:
Lại vìACCA và Alà hai tập hợprời nhaucủa tập2nC1phần tửf n; nC1; : : : ; n 1; ng nên 2nC1 D jAC CA j C j Aj chỉ có thể xảy ra khi n; n 2 A, tức là n; n 2 A. Vì n2 A, theo cấu trúc tậpAvà định nghĩa tậpAC, thì trong mỗi cặp phần tử sau đây (nchẵn)
f1; n 1g; f2; n 2g; : : : ;nn
2 1;n 2 C1o
; nn 2 o
chỉ có nhiều nhất một phần tử trong mỗi cặpthuộc vào AC. Dẫn đến jAjC 6 n
2, tương tự jA j6 n
2. Do đó
jAj D jACj C jA j 6 n 2 C n
2 Dn mâu thuẫn vớijAj DnC1. Vậy bài toán được chứng minh hoàn toàn.
Ví dụ 6 (Việt Nam TST 2012). Cho số nguyên tốp > 17. Chứng minh rằng t D 3là số nguyên dương lớn nhất thỏa mãn điều kiện: Với các số nguyên bất kỳa; b; c; d sao cho sốabc không chia hết cho p và aCb Cc chia hết chop thì tồn tại các số nguyênx; y; z thuộc tập n
0; 1; 2; : : :hp t
i 1o
sao choaxCbyCczCd:::p.
Lời giải. Nhận xét Nếu fu1; : : : ; ukg chứa một hệ thặng dư đầy đủ modulo p thì tập d C fu1; : : : ; ukgcũng chứa một hệ thặng dư đầy đủ modulop.
ĐặtkDhp t
i 1, vàAD f0; 1; 2; : : : ; kg. Đặt
S D faxCbyCczj06x; y; z 6kg D aACbACcA .mod p/:
Với t D 3, theo yêu cầu bài toán và nhận xét ở trên thìbài toán tương đương với việc chứng minh nếut D3thìS chứa một hệ thặng dư đầy đủ modulop, tức cần chứng minhjSj Dp. ĐặtC D faxCbyCczj 16x; y; z 61g:DoaCbCc 0 .mod p/
nên với mỗi bộ.x; y; z 2 A/thì
axCbyCcz a.x k/Cb.y k/Cc.z k/ .mod p/:
Tức là mỗi bộ.x; y; z/2Atương ứng với bộ.x k; y k; z k/, nên tathực hiện đối xứng tậpAđể sử dụng tậpC, bằng cách đặtLD
k 2
và xét tập S0D faxCbyCczj L6x; y; z 6Lg thì
jSj>jS0j .dấu bằng xảy ra khi và chỉ khikchẵn/:
Khi đó tậpS0viết được dưới dạng
S0DC„ CCƒ‚C CC… Llần
:
Theo định lý 2.6 ta có
jS0j>minfp; L:jCj .L 1/g: ./
Để có được kết luận bài toán làjSj Dp, ta cần chỉ rajS0j Dp, do đó theo (*) ta sẽ cần chứng minh
LjCj .L 1/>p ./:
Với chú ýa; b; ckhông chia hết chopnên tậpf0; a; b; c; a; b; c; a b; b c; c ag gồm 10 phần tử đôi một phân biệt theo modulopnằm trong tậpC. Do đójCj>10. Vậy để có (**) ta sẽ chứng minh
10L .L 1/>0,9LC1>p: . / Vìp nguyên tố, nên xét hai trường hợp
– p dạngp D 6aC1. Do p >17nêna > 3. Khi đó k D hp 3 i
1 D 2a 1, nên LDk
2
D
a 1
2
Da 1. Do đó (***) tương đương 9.a 1/C1>6aC1)a>3 .đúng/:
– p dạng p D 6aC5. Do p > 17 nên a > 2. Khi đó k D hp 3 i
1 D 2a, nên LDk
2
Da. Do đó (***) tương đương
9.a/C1>6aC5,a> 4
3 )a>2 .đúng, doanguyên/:
Vậy trong mọi trường hợp củapthì (***) đều đúng. Vậy bài toán được chứng minh trong trường hợpt D3.
Ta chứng minht D4không thỏa mãn. Thật vậy, chọnaDb D1; c D 2vàd D p 1 2 . Khi đó vớix; y; z2 AD˚
0; 1; 2; : : : ;p 4
1 thì
2hp 4 i
1
C p 1
„ ƒ‚ 2 …
xDyD0;zDŒp4 1
6xCy 2z Cp 1
2 62hp 4 i
1
Cp 1
„ ƒ‚ 2 …
xDyDŒp4 1;zD0
hay 3
2 6xCy 2zC p 1
2 6p 5
2: Trong khoảng giá trị
3
2; p 5 2
không có số nào chia hết cho p, tức không tồn tại x; y; zthỏa mãn.
Bài toán được chứng minh.
Ví dụ 7 (IMO Shortlist 2007). Cho X là tập hợp gồm 10000 số nguyên, không có số nào trong chúng chia hết cho 47. Chứng minh rằng tồn tại một tập conY củaX gồm 2007 phần tử sao choa bCc d Ce 6::: 47;8a; b; c; d; e 2Y.
Phân tích và hướng dẫn giải. Một tập M gồm các số nguyên được gọi là tốt nếu 47 6 j a bCc d Ce;8a; b; c; e; d 2 M.
Nhận thấy tập J D f 9; 7; 5; 3; 1; 1; 3; 5; 7; 9g là một tập tốt. Thật vậy, với mọi phần tửa; b; c; d; e 2J thì sốa bCc d Celà số lẻ và
45D. 9/ 9C. 9/ 9C. 9/6a bCc dCe69 . 9/C9 . 9/C9D45 nhưng không cósố nguyên lẻ nào chia hết cho 47 nằm trong tậpf 45; 43; : : : ; 43; 45g. Với mỗik D 1; 2; : : : ; 46, thì các tập hợpAk D fx 2 Xj9j 2 J W kx j .mod 47/g
đều là tập tốt. Thật vậy, giả sử tồn tại một tậpAk không tốt (k 2 f1; 2; : : : ; 46g). Khi đó tồn tại 5 phần tửx1; x2; x3; x4; x52 Ak sao cho
x1 x2Cx3 x4Cx50 .mod 47/)k.x1 x2Cx3 x4Cx5/0 .mod 47/:
Dẫn đếnkx1 kx2Ckx3 kx4Ckx5 0 .mod47/. Theo định nghĩa củax1; : : : ; x5, thì tồn tạij1; : : : ; j5 2J sao cho
j1 j2Cj3 j4Cj5 kx1 kx2Ckx3 kx4Ckx5 0 .mod 47/
chứng tỏJ là tập không tốt, mâu thuẫn.
Mỗi phần tửx 2Xthuộcđúng 10 tậpAk. Thật vậy, vì 47 là số nguyên tố, tậpf1; 2; : : : ; 46g lập thành một hệ thặng dư thu gọn modulo 47, do x 2 X nên .x; 47/ D 1, do đó fx; 2x; : : : ; 46xg cũng lập thành một hệ thặng dư thu gọn modulo 47. Do đó vớij 2 J (gồm có 10 giá trị j) thì tương ứng sẽ tồn tại 10 chỉ số k 2 f1; 2; : : : ; 46g (cácj khác nhau thì k cũng khác nhau) để kx j .mod 47/. Do đó x sẽ thuộc vào đúng 10 tập Ak.k 2 f1; 2; : : : ; 47g/.
Do đó
X4 kD1
6jAkj D10jXj D 100000:
Theo nguyên lý trung bình, tồn tại một chỉ sốksao cho jAkj> 100000
46 > 2173 > 2007;
tức tậpAk này chính là tập cần tìm.
Ta có điều phải chứng minh.
Trong lời giải trên, ta không thấy sử dụng định lý Cauchy - Davenport. Hãy nhìn kỹ lời giải trên, thì bước đầu tiên chỉ ra tậpJ là quan trọng nhất. TậpJ là tập tốt có đúng 10 phần tử.Thực sự thì tập chứa các số dư khác nhau khi chia cho 47 mà là tập tốt thì chỉ chứa tối đa 10 phần tử. Thật vậy, nếujJj>11, thì theo định lý 2.6 thì, do5jJj 4>511 4D51 > 47
jJ CJ CJ C. J /C. J /j>minf5jJj 4; 47g D47
do đó sẽ tồn tạia; b; c; d; e 2 J sao choa bCc d Ce ::: 47, suy ra tậpJ không tốt. Vậy jJj610. ĐểjJj D 10thì theo định lý 2.7,J phải là một cấp số cộng.
Ví dụ 8 (Định lý Erdos - Ginzburg - Ziv). Chon>1vàa0; a2; : : : ; a2n 2là một dãy gồm 2n 1số nguyên (không nhất thiết phân biệt). Chứng minh rằng tồn tại một dãy con gồmnsố ai1; : : : ; ainsao cho
ai1 Cai2 C Cain 0 .mod n/:
Phân tích và hướng dẫn giải. Trước tiên ta chứng minh kết luận bài toán trong trường hợp nDplàsố nguyên tố. Gọia0i 2Z; 06a0i < psao choai a0i .mod p/(thực hiện lấy modulo các phần tử của dãy). Ta có thể đánh lại chỉ số cho các sốai để
06a00 6a016: : :6a2p 20 6p 1:
– Nếuai0 a0iCp 1 với một chỉ sối 2Œ1; p 1thì theo thứ tự trên xảy ra a0i a0iC1 a0iCp 1 .mod p/
dẫn đến
ai CaiC1C CaiCp 1 pai 0 .mod p/:
– Nếuai0 a0iCp 1 với mọii 2Œ1; p 1. Khi đó đặt Ai D fai0; a0iCp 1g thìjAij D 2;8i D1; 2; : : : ; p 1. Áp dụng định lý 2.6 ta có
jA1CA2C CAp 1j>minfp;jA1jC CjAp 1j .p 2/g Dminfp; 2.p 1/ .p 2/g Dp:
Từ đây suy ra
A1CA2C CAp 1 DZp:
Từ đó tồn tại một lớp các đồng dưaj0i 2 Ai,i D1; 2; : : : ; p 1, vớiji 2 fi; iCp 1g sao cho
a0 a0j1Ca0j2C Ca0jp 1 .mod p/:
Suy ra
aj1Caj2C Cajp 1Ca00 .mod p/:
Vậy bài toán đúng trong trường hợpnnguyên tố.
Ta chứng minh kết luận bài toán trong trường hợp tổng quát bằng phương pháp quy nạp.
NếunD1, bài toán hiển nhiên đúng. Giả sử vớin > 1vàkết luận bài toán đúng cho mọi số nguyên dương nhỏ hơnn. Ta sẽ chứng minh bài toán đúng vớin. Nếunnguyên tố, bài toán đúng theo ý trên. Nếunhợp số, đặt
nDuv .1 < u6v < n/:
Khi đó theo giả thiết quy nạp, bài toán đúng chouvàv.
– Từ dãy sốa0; : : : ; a2n 2 có độ dài2n 1 D 2uv 1, theo giả thiết quy nạp, luôn tồn tại một dãy cona1;i1; : : : ; a1;iv sao cho
a1;i1C Ca1;iv 0 .mod v/:
– Khi đó còn lại2n 1 v D .2u 1/v 1số nguyên nằm trong dãy ban đầu, nhưng không thuộc vào dãy con ở trên. Do2u 1> 2, lại áp dụng giả thiết quy nạp, từ dãy độ dài .2u 1/v 1 > 2v 1 số nguyên, luôn tồn tại một dãy con a2;i1; : : : ; a2;iv sao cho
a2;i1C Ca2;iv 0 .mod v/:
– Khi đó còn lại2n 1 2v D .2u 2/v 1số nguyên trong dãy và không thuộc vào hai dãy con ở trên. Cứ tiếp tục quá trình trên, vớij D1; 2; : : : ; 2u 1, ta nhận được2u 1các dãy con đôi một rời nhauaj;i1; : : : ; aj;iv độ dàivsao cho
aj;i1C Caj 2;iv 0 .modv/:
Khi đó
aj;i1C Caj 2;iv Dbjv.bj 2Z/; 8j D1; 2; : : : ; 2u 1:
– Do giả thiết đúng chou, nên từ một dãyb1; b2; : : : ; b2u 1 độ dài2u 1sẽ có một dãy conbj1; : : : ; bju sao cho
bj1Cbj2C Cbju 0 .mod u/;
tức là
bj1Cbj2 C Cbju Dcu .c 2Z/:
Khi đó u
X
rD1
Xv sD1
ajr;is D Xu rD1
bjrv DcuvDcn0 .modn/:
Định lý được chứng minh hoàn toàn.
Ví dụ 9 (China TST 2016). Chom; nlà các số nguyên dương thỏa mãnn>m >1vàS là một tập hợp gồm cónsố tự nhiên. Chứng minh rằngS chứa ít nhất2n mC1 tập con phân biệt, mà mỗi tập đó có tổng các phần tử chia hết chom(tập rỗng xem như là tập con thỏa mãn).
Trước khi chứng minh, ta cần sử dụng một số kết quả quen thuộc cho bởi các nhận xét sau:
Nhận xét 1: Trongnsố nguyên tùy ý, luôn tồn tại một số hoặc một vài số mà tổng của chúng chia hết chon. Đây là một tính chất quen thuộc chứng minh bằng Dirichlet.
Nhận xét 2:Chop là số nguyên tố và Q D fa1; a2; : : : ; ap 1glà tập hợp chứa các số nguyên không chia hết chop. Khi đó tập
R D fS.A/ DX
x2A
xjAQg
chứa một hệ thặng dư đầy đủ modulop. Ở đâyS.;/D0:
Chứng minh. Ta có thể lấy các phần tử của Qtheo modulop mà không làm thay đổi bài toán.
Xét các tập
A1 D f0; a1g; A2 D f0; a2g; : : : ; Ap 1 D f0; ap 1g
thì doai 6::: p;8i D1; 2; : : : ; p 1nênjAij D2; i D1; 2; : : : ; p 1. Theo định lý 2.6 thì jA1CA2C CAp 1j>minfp;jA1jC CjAp 1j .p 2/g Dminfp; 2.p 1/ .p 2/g Dp:
Chứng tỏA1C CAp 1 DZp. Mặt khác, các phần tử trongA1CA2C CAp 1là các phần tử trongR(phần tử0C0C C0trongA1C CAp 1 ứng với việc chọn tậpAD ; Q).
Lời giải. Ý tưởng của bài toán là ta sẽ chia tậpS thành hai tậpA; B. Mỗi tập conA0trong A đều tìm được một tập conB0trongBmà tổng các số của hai tập con này chia hết chom. Đồng nghĩa với việcS.A0/ S.B0/ .modm/.
Trước tiên ta chứng minh nếumDpnguyên tố, bài toán hiển nhiên đúng. Thật vậy:
– Nếu trongScó ít nhấtp 1số nguyên không chia hết chom. GọiQD fa1; a2; : : : ; ap 1g là tập chứap 1số nguyên không chia hết chop. Khi đó
S DQ[.SnQ/:
TậpSnQcón .p 1/Dn pC1phần tử nên có2n pC1tập con. Mỗi tập con A.SnQ/, do nhận xét 2, tồn tại một tập conB Qsao cho
S.A/ S.B/ .mod p/)S.A/CS.B/ 0 .mod p/)S.A[B/0 .mod p/:
Vậy cứ một tập con A .SnQ/, luôn tìm được một tập dạng A[ B S để S.A[B/::: p. Mà có2n pC1 tậpAnên tương ứng có2n pC1 tập con trongS thỏa mãn đề bài.
– Nếu trong S có < p 1 số nguyên không chia hết cho m. Dẫn đến trong S có
> n pC1số chia hết cho m. Trong trường hợp này hiển nhiên mọi tập con của tập chứan pC1số này đều chia hết chop, tức là cũng có ít nhất2n pC1tập thỏa mãn bài toán.
Trong trường hợpmtùy ý. Ta chứng minh khẳng định bài toán bằng quy nạp. Vớim D1 bài toán hiển nhiên đúng. Giả sử bài toán đúng cho mọi giá trị nguyên dương< m. Xét với số nguyên dươngm. Đặtm D p:k, vớip nguyên tố vàk > 1. Một tậpA S được gọi là tập “gần tốt tối thiểu” nếu S.A/::: k nhưngS.A/ 6 :::m vàjAjnhỏ nhất. Khi đó theo nhận xét 1 thì một tậpAlàgần tốt tối thiểuthìjAj 6k.
– Nếu trong tậpS chỉ có tối đa t < p 1 tập gần tốt tối thiểu. Gọi các tập này là S1; : : : ; St. Khi đó xét phân hoạch củaS
S DS1[S2[ [St[N .N DSn.S1[S2[ [St//:
Vì mỗi tậpjSijchỉ có tối đakphần tử. Nên tậpN có
jNj>n tp > n .p 1/k Dn mCk>k:
Áp dụng giả thiết quy nạp, thì tậpN chứa
2jNj kC1 >2n mC1
tập con phân biệt chia hết chok. Tuy nhiên vì có tốt đat tậpgần tốt tối thiểu, do đó 2n mC1 tập con này mặc dù chia hết chok, không không thể là tậpgần tốt tối thiểu, tức mỗi tập con này có tổng chia chết cho m. Kết luận bài toán được chứng minh trong trường hợp này.
– Nếu trong tậpS có>p 1tậpgần tốt tối thiểu. Chọn rap 1tậpgần tốt tối thiểu phân biệt, ký hiệu làB1; : : : ; Bp 1. Khi đó xét phân hoạch củaS
S DB1[B2[ [Bp 1[M .M DSn.S1[S2[ [St//:
Vì mỗi tậpjSijchỉ có tối đakphần tử. Nên tậpM có
jMj>n .p 1/k Dn mCk>k:
Áp dụng giả thiết quy nạp, thì tậpM chứa
2jMj kC1>2n mC1
tập con phân biệt chia hết chok. Do định nghĩa củaB1; : : : ; Bp 1, đặt
S.B1/Dr1k; S.B2/Dr2; : : : ; S.Bp 1/Drp 1:k .ri 2 ZC; i D1; 2; : : : ; p 1/
và cácri 6::: p;8i D1; 2; : : : ; p 1. ĐặtQD fr1; : : : ; tp 1g, theo nhận xét 2, tập R D fS.A/jAQg
chứa một hệ thặng dư đầy đủ modulop. Do đó tập S.B1CB2C CBp 1/ sẽ chứa hệ thặng dưfk; 2k; : : : ; pkg .mod m/. Do đó lấy bất kỳ một tập conA M (khi đóS.A/::: k), luôn tồn tại một tập conB B1C CBp 1 sao cho
S.B/ S.A/ .modm/)S.A[B/0 .mod m/;
tức tậpA[Bthỏa mãn. Vì có>2n mC1tậpAM màS.A/::: k, nên có>2n mC1 tập thỏa mãn yêu cầu bài toán.
Tổng hợp các kết quả trên, bài toán được chứng minh hoàn toàn.
Lời giải cho trường hợpmDp là một trường hợp riêng cho lời giảimtổng quát. Tuy nhiên tác giả vẫn trình bày ở đây, để cho thấy cách làm với m tổng quát, dựa vào cách làm vớim D p nguyên tố.